Difference between revisions of "2024 AMC 8 Problems/Problem 12"

m (Protected "2024 AMC 8 Problems/Problem 12": Excessive spamming ([Edit=Allow only administrators] (expires 17:00, 25 January 2024 (UTC)) [Move=Allow only administrators] (expires 17:00, 25 January 2024 (UTC))))
(Problem)
Line 1: Line 1:
 
==Problem==
 
==Problem==
 +
Rohan keeps a total of 90 guppies in 4 fish tanks.
 +
 +
    <math>\bullet</math> There is 1 more guppy in the 2nd tank than in the 1st tank.
 +
    <math>\bullet</math> There are 2 more guppies in the 3rd tank than in the 2nd tank.
 +
    <math>\bullet</math> There are 3 more guppies in the 4th tank than in the 3rd tank.
 +
 +
How many guppies are in the 4th tank?
 +
 +
<math>\textbf{(A)}\ 20 \qquad \textbf{(B)}\ 21 \qquad \textbf{(C)}\ 23 \qquad \textbf{(D)}\ 24 \qquad \textbf{(E)}\ 26</math>
 +
 
==Solution 1==
 
==Solution 1==

Revision as of 15:17, 25 January 2024

Problem

Rohan keeps a total of 90 guppies in 4 fish tanks.

    $\bullet$ There is 1 more guppy in the 2nd tank than in the 1st tank.
    $\bullet$ There are 2 more guppies in the 3rd tank than in the 2nd tank.
    $\bullet$ There are 3 more guppies in the 4th tank than in the 3rd tank.

How many guppies are in the 4th tank?

$\textbf{(A)}\ 20 \qquad \textbf{(B)}\ 21 \qquad \textbf{(C)}\ 23 \qquad \textbf{(D)}\ 24 \qquad \textbf{(E)}\ 26$

Solution 1